Author Topic: EM Drive Developments - related to space flight applications - Thread 2  (Read 3322442 times)

Offline Rodal

  • Senior Member
  • *****
  • Posts: 5911
  • USA
  • Liked: 6124
  • Likes Given: 5564
Do higher em frequencies deliver increased net effect?
Excellent question. To which as yet we have no answers. This might be taken to imply that we should find out :)

But but but...there is no guarantee that these forces in any way combine to produce a larger force. And that's static I mean. I'm even more doubtful about the free space dynamics.

Statically we care about k, or N/W
Dynamically we don't know what we care about yet  8)
We have three tentative answers.  They are the mathematical formulas based on their respective theoretical models:

Quote from: John von Neumann
The sciences do not try to explain, they hardly even try to interpret, they mainly make models. By a model is meant a mathematical construct which, with the addition of certain verbal interpretations, describes observed phenomena. The justification of such a mathematical construct is solely and precisely that it is expected to work.

If a theory does not have a mathematical formula predicting variables, it doesn't qualify as a physical theory.

The predictive formulas of Shawyer and of McCulloch both are inversely proportional to the frequency.  So, on the contrary, both Shawyer and McCulloch predict that the higher the frequency the smaller the thrust force.

The formulas of Shawyer and of McCulloch have no dependence on mode shape.

@Notsosureofit's formula is the only formula that shows dependence on mode shape (through the Bessel zeros, Xmn or X'mn) and hence it has a more nuanced, subtle dependence on frequency. 

As the Bessel zeros Xmn (for TM modes) and X'mn (for TE modes) increase with frequency, @Notsosureofit's thrust force increases with frequency for mode shapes with (both) high m and n (circular and radial) quantum numbers (because Xmn and X'mn increase with frequency at a faster rate than the frequency itself)

 @Notsosureofit's thrust force decreases with frequency for mode shapes having (both) low m and n (circular and radial) quantum numbers (but in this case some of these low m and n mode shapes may be cut off at higher frequencies).


Testing experiments have not yet been performed at a large enough range of frequencies to discriminate whether any of these formulas is correct with respect to frequency variation.

In this context, performing experiments at higher microwave frequencies, perhaps using a Gunn Diode would be most helpful.
« Last Edit: 05/11/2015 12:07 pm by Rodal »

Offline CW

  • Full Member
  • *
  • Posts: 188
  • Germany
  • Liked: 141
  • Likes Given: 51
Our understanding of conservation is very deep, and Emmie Noether discovered it. She found that for every symmetry or invariance there exists a corresponding conservation law. For momentum it's the symmetry of  space. For energy it's the symmetry of time. It's actually more mathematically complex than that, since it involves differentials of the Lagrangian.
http://en.wikipedia.org/wiki/Noether%27s_theorem

Susskind has some public lectures about all the gory details, on YouTube.

That's the point I want to make. We're working with a model that has proven to be correct so far, for a century. Everybody is so used to applying this powerful insight, that perception and thought patterns are fixated on it. What, for instance, if the symmetry of space is (temporarily) being changed by a device like an EM-drive? I think that such a thing might be at least in some way outside of Noether's theorem. It might all depend on whether or not the QV is immutable and indestructible. What do you think?

BR,
-CW
Reality is weirder than fiction

Offline SeeShells

  • Senior Member
  • *****
  • Posts: 2442
  • Every action there's a reaction we try to grasp.
  • United States
  • Liked: 3186
  • Likes Given: 2708
Quote
SeeShells:
The second law of thermodynamics states that the entropy of an isolated system never decreases and the EM Chamber is an isolated enclosed system, we think.  If we are getting thrust that, thrust must be acting outside the chamber in some form. This is why I asked the simple question if smoke was used in the tests, it wasn't to detect thermal air currents but to see if it was moving away from any thrust from the EM Chamber. Smoke is small .5 to 2 um and might be be directly effected. If not then look for other forms of accelerated energy, providing thrust emanating out of the EM Chamber.

I agree with your thought train.

I have a feeling that we are so busy looking at the big paw print in the dirt that we don't see the bear sneaking up behind us.

Is it really the reflection/refraction of microwave photons that are the core issue here?.
Or is it the expanded energy shell they carry... that any high voltage source might carry?

It is obvious that the thermal state of the test device will be impacted.

Is the gravitational mass of the test unit "altered" between power up/power down?.
Is the "inertia" of the test device impacted?.
Is the Virtual mass of the test unit altering?, the reactionary energy based mass attachment to space-time. {Mach principle}

Does the chamber emit external acoustic waves? ( well beyond the human hearing range)
Nice rundown of most everything that can change the energy state of the Em Chamber at rest. I believe (unless new physics is discovered here, then the whole ballgame changes) whatever is happening it's going to be interesting. This rundown was the site that got me thinking.
http://hyperphysics.phy-astr.gsu.edu/hbase/quantum/phodens.html#c1
Photon Energy Density


Offline deltaMass

  • Full Member
  • ****
  • Posts: 955
  • A Brit in California
  • Liked: 671
  • Likes Given: 275
@arc:
Quote
Do higher em frequencies deliver increased net effect?
Excellent question. To which as yet we have no answers. This might be taken to imply that we should find out :)

But but but...there is no guarantee that these forces in any way combine to produce a larger force. And that's static I mean. I'm even more doubtful about the free space dynamics.

Statically we care about k, or N/W
Dynamically we don't know what we care about yet  8)
We have three tentative answers.  They are the mathematical formulas based on their respective theoretical models:

Quote from: John von Neumann
The sciences do not try to explain, they hardly even try to interpret, they mainly make models. By a model is meant a mathematical construct which, with the addition of certain verbal interpretations, describes observed phenomena. The justification of such a mathematical construct is solely and precisely that it is expected to work.

If a theory does not have a mathematical formula predicting variables, it doesn't qualify as a physical theory.

The predictive formulas of Shawyer and of McCulloch both are inversely proportional to the frequency.  So, on the contrary, both Shawyer and McCulloch predict that the higher the frequency the smaller the thrust force.

The formulas of Shawyer and of McCulloch have no dependence on mode shape.

@Notsosureofit's formula is the only formula that shows dependence on mode shape (through the Bessel zeros, Xmn or X'mn) and hence it has a more nuanced, subtle dependence on frequency. 

As the Bessel zeros Xmn (for TM modes) and X'mn (for TE modes) increase with frequency, @Notsosureofit's thrust force increases with frequency for mode shapes with (both) high m and n (circular and radial) quantum numbers (because Xmn and X'mn increase with frequency at a faster rate than the frequency itself)

 @Notsosureofit's thrust force decreases with frequency for mode shapes having (both) low m and n (circular and radial) quantum numbers (but in this case some of these low m and n mode shapes may be cut off at higher frequencies).

That's a nice enumeration. But what of Sonny White's PV derivative theory - called QVF I believe?

Offline deltaMass

  • Full Member
  • ****
  • Posts: 955
  • A Brit in California
  • Liked: 671
  • Likes Given: 275
Our understanding of conservation is very deep, and Emmie Noether discovered it. She found that for every symmetry or invariance there exists a corresponding conservation law. For momentum it's the symmetry of  space. For energy it's the symmetry of time. It's actually more mathematically complex than that, since it involves differentials of the Lagrangian.
http://en.wikipedia.org/wiki/Noether%27s_theorem

Susskind has some public lectures about all the gory details, on YouTube.

That's the point I want to make. We're working with a model that has proven to be correct so far, for a century. Everybody is so used to applying this powerful insight, that perception and thought patterns are fixated on it. What, for instance, if the symmetry of space is (temporarily) being changed by a device like an EM-drive? I think that such a thing might be at least in some way outside of Noether's theorem. It might all depend on whether or not the QV is immutable and indestructible. What do you think?

BR,
-CW
I have a real problem believing that an oddly-shaped cavity is going to change the workings of spacetime such that translational invariance is affected. But it's a creative suggestion nonetheless. On the other hand, if the thrust is truly not a measurement artifact, I have no explanation for it.

Offline Rodal

  • Senior Member
  • *****
  • Posts: 5911
  • USA
  • Liked: 6124
  • Likes Given: 5564
...
That's a nice enumeration. But what of Sonny White's PV derivative theory - called QVF I believe?
I have never seen an explicit mathematical expression for Dr. White's QVF thrust force in any of his papers.

Lately a new computer program of Dr. White was described and an image posted, described as displaying dependence on FM, AM and PM modulation, but the mathematical expressions on what the program was based were not released to the public.

I have read his papers (NOTE TO ALL READERS: please no need to bring up papers that talk about the QVF, with respect to this message, if they do not contain a mathematical expression explicitly showing a dependence of thrust force with frequency  ;)   )
« Last Edit: 05/11/2015 12:16 pm by Rodal »

Offline marshallC

  • Member
  • Posts: 10
  • Liked: 2
  • Likes Given: 25

Interesting concept, I like it. 
10 stacks of "cavities" 100 units high arranged either side by side (flat/wall) or in a circular/tubular form. The tube would allow you to feed the energy up the middle to each drive unit.. 1000 micro-thrusters in Series-Parallel
Sounds like a 3D printing job. Glass would not stand that heat for very long... perhaps Silver?. Or Titanium with polished silver coating

Each cavity could use the wall of its neighbor for mechanical rigidity and the gaps resulting from conic shaped chambers would allow a coolant to be pumped around the cavities. Drive the units in cyclic pumped mode to minimise thermal overloading.

In this setup, would we be able to use one light source that's split up between all of the parallel units, or would we have individual light sources for each stack?  If I understand correctly, the force observed is far more than even a perfect photon thruster, so perhaps the potential to lose energy or get conflicting thrust while redirecting photos from one source into each individual frustum might not be significant compared to the minituarization it could allow compared to having multiple sources.


Offline MrVibrating

  • Member
  • Posts: 5
  • London UK
  • Liked: 9
  • Likes Given: 5
Our understanding of conservation is very deep, and Emmie Noether discovered it. She found that for every symmetry or invariance there exists a corresponding conservation law. For momentum it's the symmetry of  space. For energy it's the symmetry of time. It's actually more mathematically complex than that, since it involves differentials of the Lagrangian.
http://en.wikipedia.org/wiki/Noether%27s_theorem

Susskind has some public lectures about all the gory details, on YouTube.

That's the point I want to make. We're working with a model that has proven to be correct so far, for a century. Everybody is so used to applying this powerful insight, that perception and thought patterns are fixated on it. What, for instance, if the symmetry of space is (temporarily) being changed by a device like an EM-drive? I think that such a thing might be at least in some way outside of Noether's theorem. It might all depend on whether or not the QV is immutable and indestructible. What do you think?

BR,
-CW

It's not so much symmetry of space (or time) that Noether's theorem concerns, but rather the fields within and the interactions possible with their forces.  So for example an Alcubierre drive doesn't transgress the principle - the physics within the bubble remaining entirely consistent with it.  Rather than viewing a symmetry break as a Noether exception, it's more fruitful to consider how it works within the framework. 

As Frobnicat alluded  a few posts back, any apparent violation merely indicates the system is open, albeit unexpectedly so.  One possible way of negotiating around the constrainsts is to play off two different interactions against each other, wherein both interactions are fully conservative in their own right, but the exchange of energy between them is subject to time-dependent variations in one or other, resulting in an open system from the interplay between two closed ones.  I gave an example a few pages back of such a non-dissipative non-conservative EM exchange, however in reality it's dissipatiting energy into the vacuum.

If the EM drive is doing something similar, it would instead be drawing energy from the vacuum, so Noether, CoE and CoM would still all be satisfied...

ETA:  or rather, momentum from the vacuum - Shawyer states clearly that his drive fully conforms to CoE, so draws onboard power equal to any work done.
« Last Edit: 05/11/2015 12:36 pm by MrVibrating »

Offline Notsosureofit

  • Full Member
  • ****
  • Posts: 691
  • Liked: 747
  • Likes Given: 1729
Do higher em frequencies deliver increased net effect?
Excellent question. To which as yet we have no answers. This might be taken to imply that we should find out :)

But but but...there is no guarantee that these forces in any way combine to produce a larger force. And that's static I mean. I'm even more doubtful about the free space dynamics.

Statically we care about k, or N/W
Dynamically we don't know what we care about yet  8)
We have three tentative answers.  They are the mathematical formulas based on their respective theoretical models:

Quote from: John von Neumann
The sciences do not try to explain, they hardly even try to interpret, they mainly make models. By a model is meant a mathematical construct which, with the addition of certain verbal interpretations, describes observed phenomena. The justification of such a mathematical construct is solely and precisely that it is expected to work.

If a theory does not have a mathematical formula predicting variables, it doesn't qualify as a physical theory.

The predictive formulas of Shawyer and of McCulloch both are inversely proportional to the frequency.  So, on the contrary, both Shawyer and McCulloch predict that the higher the frequency the smaller the thrust force.

The formulas of Shawyer and of McCulloch have no dependence on mode shape.

@Notsosureofit's formula is the only formula that shows dependence on mode shape (through the Bessel zeros, Xmn or X'mn) and hence it has a more nuanced, subtle dependence on frequency. 

As the Bessel zeros Xmn (for TM modes) and X'mn (for TE modes) increase with frequency, @Notsosureofit's thrust force increases with frequency for mode shapes with (both) high m and n (circular and radial) quantum numbers (because Xmn and X'mn increase with frequency at a faster rate than the frequency itself)

 @Notsosureofit's thrust force decreases with frequency for mode shapes having (both) low m and n (circular and radial) quantum numbers (but in this case some of these low m and n mode shapes may be cut off at higher frequencies).


Testing experiments have not yet been performed at a large enough range of frequencies to discriminate whether any of these formulas is correct with respect to frequency variation.

In this context, performing experiments at higher microwave frequencies, perhaps using a Gunn Diode would be most helpful.


Really hadn't thought about it, but at large X, isn't f prop X ?  and NT prop X^2/f^3  ??

Offline Rodal

  • Senior Member
  • *****
  • Posts: 5911
  • USA
  • Liked: 6124
  • Likes Given: 5564
...
Really hadn't thought about it, but at large X, isn't f prop X ?  and NT prop X^2/f^3  ??
My recollection (from running numerical examples at the time at which you were planning to run experiments with a Gunn diode) is that at large Xmn, with both m~n simultaneously highest, Xmn increases higher than f, so that 

Xmn (f) ~ f ^y  where y>1

Need y>3/2 in order for thrust force to increase with frequency.

My recollection is that the thrust force predictions using your formula for the Gunn Diode frequency were much higher than the predictions of Shawyer and McCulloch.

EDIT: Also the calculations for Shawyer's Flight Thruster (which I recall was run at higher frequency: 3.85GHz, twice the frequency of NASA's EM Drive tests)

Is my memory correct ?  (too bad that we don't have a good search function to look for things like that  :(  )
« Last Edit: 05/11/2015 01:03 pm by Rodal »

Offline deltaMass

  • Full Member
  • ****
  • Posts: 955
  • A Brit in California
  • Liked: 671
  • Likes Given: 275
It occurs to me to ask a history question about all this. What on Earth occasioned Shawyer in the first place to carefully measure thrust on this odd-shaped cavity?

Offline TheTraveller

It occurs to me to ask a history question about all this. What on Earth occasioned Shawyer in the first place to carefully measure thrust on this odd-shaped cavity?
Did read Shawyer, back in his sat days, noticed unexpected sat movements when certain on board microwave systems were powered on which could not be explained via conventional expectations.

He did investigations & came up with his theory of what was happening. As I undersrand it, his employers were not interested, he resigned and set up SPR.

Here SPR claims seven reviews got positive results:

Quote
The team claims to have undergone seven independent reviews from experts at BAE Systems, EADS Astrium, Siemens and the IEE. The DTI has awarded the company £125,000 to develop a prototype engine as part of a three-year, £250,000 programme.

Read more: http://www.theengineer.co.uk/news/defying-gravity/266633.article#ixzz3Zq1ARpVP
« Last Edit: 05/11/2015 01:56 pm by TheTraveller »
It Is Time For The EmDrive To Come Out Of The Shadows

Offline Rodal

  • Senior Member
  • *****
  • Posts: 5911
  • USA
  • Liked: 6124
  • Likes Given: 5564
It occurs to me to ask a history question about all this. What on Earth occasioned Shawyer in the first place to carefully measure thrust on this odd-shaped cavity?
Isn't the answer given by the statements Shawyer makes in http://forum.nasaspaceflight.com/index.php?action=dlattach;topic=36313.0;attach=829636 ? , particularly his claim that

Quote from: Shawyer
In a cavity with the correct radius spherical end plates there is no force on the side walls due to the travelling wave, because the walls are parallel to the group velocity vector

This claim that there are no forces on the side walls is contradicted by a calculation of Maxwell's stress tensor, particularly under classical electromagnetism, as explicitly shown by Greg Egan  (http://gregegan.customer.netspace.net.au/SCIENCE/Cavity/Cavity.html ) for example for such a truncated cone with spherical ends.  Shawyer (to my knowledge) has never adequately answered Greg Egans's proof.

However, if one believes (as Shawyer does) that there are no forces on the conical side walls, then Shawyer's construction immediately follows.

NOTE: To my knowledge, no closed cavity truncated cones (as in Shawyer's EM Drive) are used for microwave communication satellites.
« Last Edit: 05/11/2015 01:56 pm by Rodal »

Offline Notsosureofit

  • Full Member
  • ****
  • Posts: 691
  • Liked: 747
  • Likes Given: 1729
...
Really hadn't thought about it, but at large X, isn't f prop X ?  and NT prop X^2/f^3  ??
My recollection (from running numerical examples at the time at which you were planning to run experiments with a Gunn diode) is that at large Xmn, with both m~n simultaneously highest, Xmn increases higher than f, so that 

Xmn (f) ~ f ^y  where y>1

Need y>3/2 in order for thrust force to increase with frequency.

My recollection is that the thrust force predictions using your formula for the Gunn Diode frequency were much higher than the predictions of Shawyer and McCulloch.

EDIT: Also the calculations for Shawyer's Flight Thruster (which I recall was run at higher frequency: 3.85GHz, twice the frequency of NASA's EM Drive tests)

Is my memory correct ?  (too bad that we don't have a good search function to look for things like that  :(  )

Can't remember either, that may have been before I did the units check and noticed the 1/f error (typo on post-it notes) in N (number of photons).

I should go back and set up a graphing program for these formulas.  Cavity dimensions scale as 1/f just to add to the mess.   

I never did get back to the cylindrical w/ dielectric case.   Except for a uniform change, most dielectric surfaces need to include a reflection and you need an iterative program to solve them numerically.  (wrote some of these for optical filter companies in the dim dark past)   It would be nice to have an integral solution that could be optimized.

Just a note:  CoM from spacial symmetry is thought to be local from present cosmology, or so I believe, really haven't checked.

« Last Edit: 05/11/2015 01:39 pm by Notsosureofit »

Offline rfmwguy

  • EmDrive Builder (retired)
  • Senior Member
  • *****
  • Posts: 2205
  • Liked: 2713
  • Likes Given: 1134
Bad humor alert: If someone comes up with a formula that makes the ideal frustum look like Die Glocke, I'm outta here. ;)

Offline WarpTech

  • Full Member
  • ****
  • Posts: 1407
  • Do it!
  • Statesville, NC
  • Liked: 1453
  • Likes Given: 1925
It occurs to me to ask a history question about all this. What on Earth occasioned Shawyer in the first place to carefully measure thrust on this odd-shaped cavity?
Isn't the answer given by the statements Shawyer makes in http://forum.nasaspaceflight.com/index.php?action=dlattach;topic=36313.0;attach=829636 ? , particularly his claim that

Quote from: Shawyer
In a cavity with the correct radius spherical end plates there is no force on the side walls due to the travelling wave, because the walls are parallel to the group velocity vector

This claim that there are no forces on the side walls is contradicted by a calculation of Maxwell's stress tensor, particularly under classical electromagnetism, as explicitly shown by Greg Egan  (http://gregegan.customer.netspace.net.au/SCIENCE/Cavity/Cavity.html ) for example for such a truncated cone with spherical ends.  Shawyer (to my knowledge) has never adequately answered Greg Egans's proof.

However, if one believes (as Shawyer does) that there are no forces on the conical side walls, then Shawyer's construction immediately follows.

NOTE: To my knowledge, no closed cavity truncated cones (as in Shawyer's EM Drive) are used for microwave communication satellites.


Correct. The group velocity is limited precisely because, there is EM drag on the walls of the waveguide.


Offline Rodal

  • Senior Member
  • *****
  • Posts: 5911
  • USA
  • Liked: 6124
  • Likes Given: 5564
...
Quote
The team claims to have undergone seven independent reviews from experts at BAE Systems, EADS Astrium, Siemens and the IEE. The DTI has awarded the company £125,000 to develop a prototype engine as part of a three-year, £250,000 programme.

Read more: http://www.theengineer.co.uk/news/defying-gravity/266633.article#ixzz3Zq1ARpVP

See how Shawyer's explanation is quoted (Bold and Color added for emphasis) in that article, particularly the claim that the electromagnetic wave is travelling at a speed of only 0.1 c at the small end of the EM Drive: !

Quote
At the wider end of the cone the wave travels at the speed of light, while at the other the wave travels at one tenth of that speed, due to the geometry of the waveguide.

This creates higher radiation pressure at the wider end of the waveguide because the rate of change of momentum of the waves is different. Newton’s second law defines force as the rate of change of momentum.

Shawyer explained that if these forces were the result of a working fluid, there would merely be a mechanical strain in the waveguide walls. But as the working fluid is replaced by an electromagnetic wave at close to the speed of light, Newtonian mechanics are replaced with the special theory of relativity.

’The electromagnetic wave is going at very high velocities, so you have to apply two different reference planes,’ he said. ’It can no longer be considered a closed system. As soon as you approach the speed of light the wave can be considered completely independent of the waveguide.’ Shawyer compared the engine to a laser gyroscope, which also relies on Einstein’s laws, where attitude information is obtained from an apparently closed system.

In a ring laser gyroscope, two counter-rotating beams are channeled to a photo detector. If the spaceship is not rotating, the beams remain in phase. If rotation is occurring, one beam continuously changes phase with respect to the other. A diode translates that moving interference pattern into digital pulses, each pulse representing an angle of rotation. The rate at which the pulses are produced is also a measure of the rate of rotation.



How can the principle of operation of the ring laser gyroscope be compared to the principle of operation of EM Drive thrust, escapes me...
« Last Edit: 05/11/2015 03:19 pm by Rodal »

Offline Notsosureofit

  • Full Member
  • ****
  • Posts: 691
  • Liked: 747
  • Likes Given: 1729
Just another idle thought while trying to jog my memory:

If evanescent waves turn out to be a coupling mechanism (to the universe by virtue of FTL propagation ???) then high-Q dielectric resonators have a big advantage over metallic chambers.  Such coupling was used in cold-war radars w/ spherical resonators for stability reasons.  (very low coupling constant, etc.  somewhere I probably still have a few Russian ones in a drawer)

As long as I'm rambling far afield:  Any thoughts on how to incorporate the super-regeneration effect into the system.
 That would mean pulsed (quenched) oscillation which, if anything, might detect interfering back ground signals. ???

« Last Edit: 05/11/2015 03:08 pm by Notsosureofit »

Offline Blaine

  • Member
  • Posts: 58
  • Spring Hill, KS
  • Liked: 45
  • Likes Given: 122
It's unclear from these forum posts if the prototype propulsion system actually generated any thrust during the recent tests, said Ethan Siegel, a physics and astronomy professor at Lewis & Clark College in Portland, Oregon
(http://www.space.com/29363-impossible-em-drive-space-engine-nasa.html)

Basically a new article that I like for the most part as its not too optimistic, but not too pessimistic either.  However the part that I quoted above is clearly mistaken.  As thrust was generated and is shown on this site and the Nasa test results.
Weird Science!

Offline Flyby

  • Full Member
  • ***
  • Posts: 388
  • Belgium
  • Liked: 451
  • Likes Given: 48
I'm wondering, in the context of dr. rodal's remark on Shawyer's theory, where he (Shawyer) poses that forces are to be observed on the ends only, what would happen if one or both ends were to be replace by Cullen type ring reflector or mesh?
Would one still observe forces of the same magnitude?

From what I understood, waves would still be bouncing yet the surface upon which a force can be applied, or a momentum be transferred to would be drastically reduced...

Somewhere , somehow, if this device really works, there must be a momentum transfer onto the frustum....

If it is uncertain what role the end plates play, why try dielectric materials, or materials with a high magnetic permeability (this subject got completely lost in the current discussion?) on the side walls of the frustum to see if any drag effect is in effect?

Tags:
 

Advertisement NovaTech
Advertisement Northrop Grumman
Advertisement
Advertisement Margaritaville Beach Resort South Padre Island
Advertisement Brady Kenniston
Advertisement NextSpaceflight
Advertisement Nathan Barker Photography
1